Inverse Laplace transform (Initial Value Problem)

In summary, the person is stuck trying to find the inverse Laplace of a given equation to get y(t), but is having trouble because the denominator is all squared. They have tried using a translation theorem and partial fractions, but have not been successful. They are looking for suggestions and have not yet shown their work. However, they eventually solve the problem by using a formula from the Laplace transform tables.
  • #1
aero_zeppelin
86
0

Homework Statement



I'm stuck trying to find out the inverse Laplace of F(s) to get y(t) (the solution for the differential equation):

Y(s) = 1 / [ (s-1)^2 + 1 ]^2



The Attempt at a Solution



I tried using a translation theorem and then apply the sine formula, but the denominator is still all squared. I also tried partial fractions to expand Y(s) but I didn't get it right...

Any suggestions please?
 
Physics news on Phys.org
  • #2
aero_zeppelin said:

Homework Statement



I'm stuck trying to find out the inverse Laplace of F(s) to get y(t) (the solution for the differential equation):

Y(s) = 1 / [ (s-1)^2 + 1 ]^2



The Attempt at a Solution



I tried using a translation theorem and then apply the sine formula, but the denominator is still all squared. I also tried partial fractions to expand Y(s) but I didn't get it right...

Any suggestions please?

Yes: show us what you did and what you got. How can we possibly help if we have no idea of what your problem is?
 
  • #3
[itex][(s-1)+1]^2=[(s-1+i)(s-1-i)]^2[/itex]

Therefore, [tex]\left[\frac{1}{ (s-1-i)(s-1+i) }\right]^2=\left[\frac{1}{2i}\left(\frac{1}{s-1-i}-\frac{1}{s-1+i} \right ) \right ]^2[/tex]

Can you continue it from there?
 
  • Like
Likes 1 person
  • #4
Ray Vickson said:
Yes: show us what you did and what you got. How can we possibly help if we have no idea of what your problem is?


My friend, that's the thing, I can't get past that. I stated the problem clearly: "I'm stuck trying to find out the INVERSE LAPLACE of the given Y(s) to get y(t)". There's no point in showing what I did before, it's not needed for what's after the equation I posted.

Anyway, I got it. It was just applying one of the formulas from the Laplace transform tables. I thought partial fractions or something else had to be done.

Thanks for your help
 

Related to Inverse Laplace transform (Initial Value Problem)

1. What is the inverse Laplace transform?

The inverse Laplace transform is a mathematical operation that takes a function in the Laplace domain and converts it back to the original function in the time domain. It is the inverse operation of the Laplace transform and is used to solve initial value problems in differential equations.

2. How is the inverse Laplace transform performed?

The inverse Laplace transform is typically performed using a table of known Laplace transforms, which can be found in most mathematics textbooks. It involves manipulating the Laplace domain function using algebraic and calculus techniques to transform it back into the time domain.

3. What is an initial value problem?

An initial value problem is a type of differential equation that involves finding the behavior of a function over time, given a set of initial conditions. These conditions usually include the value of the function at a specific time and its derivative at that time.

4. Can the inverse Laplace transform be used to solve any initial value problem?

No, the inverse Laplace transform can only be used to solve initial value problems that have a corresponding Laplace transform. Some initial value problems may not have a Laplace transform and therefore cannot be solved using this method.

5. Are there any limitations or drawbacks to using the inverse Laplace transform?

One limitation of using the inverse Laplace transform is that it can be difficult to compute for complex functions. In addition, the solution may not always be unique, meaning there can be multiple functions that satisfy the initial value problem. It is also important to check for convergence and validity of the solution when using the inverse Laplace transform.

Similar threads

  • Calculus and Beyond Homework Help
Replies
10
Views
1K
  • Calculus and Beyond Homework Help
Replies
2
Views
923
  • Calculus and Beyond Homework Help
Replies
4
Views
1K
  • Calculus and Beyond Homework Help
Replies
9
Views
1K
  • Calculus and Beyond Homework Help
Replies
7
Views
857
  • Calculus and Beyond Homework Help
Replies
2
Views
1K
  • Calculus and Beyond Homework Help
Replies
3
Views
2K
  • Calculus and Beyond Homework Help
Replies
8
Views
2K
  • Calculus and Beyond Homework Help
Replies
4
Views
2K
  • Calculus and Beyond Homework Help
Replies
1
Views
689
Back
Top